+ All Categories
Home > Documents > 11 x09 Capital Budgeting

11 x09 Capital Budgeting

Date post: 14-Apr-2015
Category:
Upload: aisah-dimangandam
View: 476 times
Download: 23 times
Share this document with a friend
64
Capital Budgeting MODULE 9 CAPITAL BUDGETING THEORIES: Basic Concepts Decision Making Process 2. The first step in the decision-making process is to A. determine and evaluate possible courses of action. B. identify the problem and assign responsibility. C. make a decision. D. review results of the decision. Strategic planning 39. Strategic planning is the process of deciding on an organization’ A. minor programs and the approximate resources to be devoted to them B. major programs and the approximate resources to be devoted to them C. minor programs prior to consideration of resources that might be needed D. major programs prior to consideration of resources that might be needed Capital budgeting defined 1. The long-term planning process for making and financing investments that affect a company’s financial results over a number of years is referred to as A. capital budgeting C. master budgeting B. strategic planning D. long-range planning 3. Capital budgeting is the process A. used in sell or process further decisions. B. of determining how much capital stock to issue C. of making capital expenditure decisions D. of eliminating unprofitable product line 5. A capital investment decision is essentially a decision to: A. exchange current assets for current liabilities. B. exchange current cash outflows for the promise of receiving future cash inflows. C. exchange current cash flow from operating activities for future cash inflows from investing activities. D. exchange current cash inflows for future cash outflows. Risk & return 6. The higher the risk element in a project, the A. more attractive the investment is. B. higher the net present value is. C. higher the cost of capital is. D. higher the discount rate is. 9. Cost of capital is the A. amount the company must pay for its plant assets. B. dividends a company must pay on its equity securities. C. cost the company must incur to obtain its capital resources. D. cost the company is charged by investment bankers who handle the issuance of equity or long-term debt securities. 483
Transcript
Page 1: 11 x09 Capital Budgeting

Capital Budgeting

MODULE 9

CAPITAL BUDGETING

THEORIES:Basic ConceptsDecision Making Process2. The first step in the decision-making process is to

A. determine and evaluate possible courses of action.B. identify the problem and assign responsibility.C. make a decision.D. review results of the decision.

Strategic planning39. Strategic planning is the process of deciding on an organization’

A. minor programs and the approximate resources to be devoted to themB. major programs and the approximate resources to be devoted to themC. minor programs prior to consideration of resources that might be neededD. major programs prior to consideration of resources that might be needed

Capital budgeting defined1. The long-term planning process for making and financing investments that affect a company’s

financial results over a number of years is referred to asA. capital budgeting C. master budgetingB. strategic planning D. long-range planning

3. Capital budgeting is the processA. used in sell or process further decisions.B. of determining how much capital stock to issueC. of making capital expenditure decisionsD. of eliminating unprofitable product line

5. A capital investment decision is essentially a decision to:A. exchange current assets for current liabilities.B. exchange current cash outflows for the promise of receiving future cash inflows.C. exchange current cash flow from operating activities for future cash inflows from investing

activities.D. exchange current cash inflows for future cash outflows.

Risk & return6. The higher the risk element in a project, the

A. more attractive the investment is.B. higher the net present value is.C. higher the cost of capital is.D. higher the discount rate is.

9. Cost of capital is theA. amount the company must pay for its plant assets.B. dividends a company must pay on its equity securities.C. cost the company must incur to obtain its capital resources.D. cost the company is charged by investment bankers who handle the issuance of equity or

long-term debt securities.

14. How should the following projects be listed in order of increasing risk?A. New venture, replacement, expansion.B. Replacement, new venture, expansion.C. Replacement, expansion, new venture.D. Expansion, replacement, new venture.

41. Problems associated with justifying investments in high-tech projects often include discount rates that are tooA. low and time horizons that are too longB. high and time horizons that are too longC. high and time horizons that are too shortD. low and time horizons that are too short

60. In evaluating high-tech projects,A. only tangible benefits should be considered.B. only intangible benefits should be considered.C. both tangible and intangible benefits should be considered.D. neither tangible nor intangible benefits should be considered.

Types of capital projects4. A project that when accepted or rejected will not affect the cash flows of another project.

A. Independent projects C. Mutually exclusive projectsB. Dependent projects D. Both b and c

Capital budgeting process7. The normal methods of analyzing investments

A. cannot be used by not-for-profit entities.B. do not apply if the project will not produce revenues.C. cannot be used if the company plans to finance the project with funds already available

483

Page 2: 11 x09 Capital Budgeting

Capital Budgeting

internally.D. require forecasts of cash flows expected from the project.

InvestmentsSale of old asset38. When disposing of an old asset and replacing it with a new one, tax effect on

A. gain on sale of the old asset reduces the basis of the new assetB. gain on sale of the old asset increases the basis of the new assetC. loss on sale of the old asset reduces the basis of the new assetD. b and c

Working capital18. A major difference between an investment in working capital and one in depreciable assets is

thatA. an investment in working capital is never returned, while most depreciable assets have

some residual value.B. an investment in working capital is returned in full at the end of a project’s life, while an

investment in depreciable assets has no residual value.C. an investment in working capital is not tax-deductible when made, nor taxable when

returned, while an investment in depreciable assets does allow tax deductions.D. because an investment in working capital is usually returned in full at the end of the

project’s life, it is ignored in computing the amount of the investment required for the project.

30. The proper treatment of an investment in receivables and inventory is toA. ignore itB. add it to the required investment in fixed assetsC. add it to the required investment in fixed assets and subtract it from the annual cash flowsD. add it to the investment in fixed assets and add the present value of the recovery to the

present value of the annual cash flows

31. In connection with a capital budgeting project, an investment in working capital is normally recoveredA. at the end of the project’s lifeB. in the first year of the project’s lifeC. evenly through the project’s lifeD. when the company goes out of businessA

32. XYZ Co. is adopting just-in-time principles. When evaluating an investment project that would reduce inventory, how should XYZ treat the reduction?A. Ignore it.

B. Decrease the cost of the investment and decrease cash flows at the end of the project’s life.

C. Decrease the cost of the investment.D. Decrease the cost of the investment and increase the cash flow at the end of the project’s

life.

Relevant cash flows72. Which of the following represents the biggest challenge in the decision to purchase new

equipment?A. Estimating employee training for the new project.B. Estimating cash flows for the future.C. Estimating transportation costs of the new equipment.D. Estimating maintenance costs for the new equipment.

51. When a firm has the opportunity to add a project that will utilize factory capacity that is currently not being used, which costs should be used to determine if the added project should be undertaken?A. Opportunity costs C. Net present costsB. Historical costs D. Incremental costs

11. The only future costs that are relevant to deciding whether to accept an investment are those that willA. be different if the project is accepted rather than rejected.B. be saved if the project is accepted rather than rejected.C. be deductible for tax purposes.D. affect net income in the period that they are incurred.

Cash inflow66. Which of the following is not a typical cash inflow in capital investment decisions?

A. Incremental revenues C. Salvage valueB. Cost reductions D. Additional working capital

Out-of-pocket costs45. Which of the following is a cost that requires a future outlay of cash that is which relevant for

future decision-making?A. Opportunity cost C. Sunk costsB. Out-of-pocket cost D. Relevant benefits

Depreciation & Tax 22. If there were no income taxes,

A. depreciation would be ignored in capital budgeting.

484

Page 3: 11 x09 Capital Budgeting

Capital Budgeting

B. the NPV method would not work.C. income would be discounted instead of cash flow.D. all potential investments would be desirable.

21. Relevant cash flows for net present value (NPV) models include all of the following exceptA. outflows to purchase new equipmentB. depreciation expense on the newly acquired piece of equipmentC. reductions in operating cash flows as a result of using the new equipment.D. cash outflows related to purchasing additional inventories for another retail store.

55. When evaluating depreciation methods, managers who are concerned about capital investment decisions will:A. choose straight line depreciation so there is minimum impact on the decision.B. use units of production so more depreciation expense will be allocated to the later years.C. use accelerated methods to have as much depreciation in the early years of an asset’s

life.D. choice of depreciation method has no impact on the capital investment decision.

70. The tax consequences should be considered under which circumstances when making capital investment decisions?A. Positive net income C. DepreciationB. Disposal of an asset D. All of the above

Irrelevant cash flowsLoan financing43. In addition to incremental revenues, cash inflows from capital investments can be generated

from all of the following sources except:A. debt financingB. cost savingsC. salvage valueD. reduction in the amount of working capital

10. If Helena Company expects to get a one-year bank loan to help cover the initial financing of one of its capital projects, the analysis of the project shouldA. offset the loan against any investment in inventory or receivables required by the project.B. show the loan as an increase in the investment.C. show the loan as a cash outflow in the second year of the project’s life.D. ignore the loan

Sunk cost29. In deciding whether to replace a machine, which of the following is NOT a sunk cost?

A. The expected resale price of the existing machine.B. The book value of the existing machine.C. The original cost of the existing machine.D. The depreciated cost of the existing machine.

Accounting rate of return54. The primary advantages of the average rate of return method are its ease of computation and

the fact that:A. It is especially useful to managers whose primary concern is liquidityB. There is less possibility of loss from changes in economic conditions and obsolescence

when the commitment is short-termC. It emphasizes the amount of income earned over the life of the proposalD. Rankings of proposals are necessary

Nondiscounted cash flow methodPayback method36. There are several capital budgeting decision models that do not use discounted cash flows.

What is the name of the simple technique that calculates the total time it will take to recover, using cash inflows from operations, the amount of cash invested in a project?A. Recovery period C. External rate of returnB. Payback model D. Accounting rate of return

34. The technique most concerned with liquidity isA. Payback method.B. Net present value technique.C. Internal rate of return.D. book rate of return.

73. Which of the following is a potential use of the payback method?A. Help managers control the risks of estimating cash flowsB. Help minimize the impact of the investment on liquidityC. Help control the risk of obsolescenceD. All of the answers are correct

47. The cash payback technique:A. should be used as a final screening tool.B. can be the only basis for the capital budgeting decision.C. is relatively easy to compute and understand.D. considers the expected profitability of a project.

33. Which of the following is NOT a defect of the payback method?

485

Page 4: 11 x09 Capital Budgeting

Capital Budgeting

A. It ignores cash flows because it uses net income.B. It ignores profitability.C. It ignores the present values of cash flows.D. It ignores the pattern of cash flows beyond the payback period.

48. The payback method, as a capital budgeting technique, assumes that all intermediate cash inflows are reinvested to yield a return equal to:A. Zero C. The Discount RateB. The Time-Adjusted-Rate-of-Return D. The Cost-of-Capital

52. Which of the following capital budgeting methods is the least theoretically correct?A. payback method C. internal rate of returnB. net present value D. none of the above

Discounted cash flow method49. Which of the following methods of evaluating capital investment projects incorporates the time

value of money?A. Payback period, accounting rate of return, and internal rate of returnB. Accounting rate of return, net present value, and internal rate of returnC. Payback period and accounting rate of returnD. Net present value and internal rate of return

Net present value69. Discounted cash flow analysis is used in which of the following techniques?

A. Net present value C. Cost of capitalB. Payback period D. All of the above

8. The primary capital budgeting method that uses discounted cash flow techniques is theA. net present value method.B. cash payback technique.C. annual rate of return method.D. profitability index method.

20. The net present value (NPV) model can be used to evaluate and rank two or more proposed projects. The approach that computes the total impact on cash flows for each option and then converts these total cash flows to their present values is called theA. differential approach C. contribution approachB. incremental approach. D. total project approach.

40. The discount rate commonly used in present value calculations is theA. treasury bill rate

B. weighted average return on assets adjusted for riskC. risk free rate plus inflation rateD. shareholders’ expected return on equity

44. Which is true of the net present value method of determining the acceptability of an investment?A. The initial cost of the investment is subtracted from the present value of net cash flowsB. The net cash flows are not adjusted to present valueC. A negative net present value indicates the investment should be undertakenD. The net present value method requires no subjective judgments

Profitability index35. The profitability index

A. does not take into account the discounted cash flows.B. Is calculated by dividing total cash flows by the initial investment.C. allows comparison of the relative desirability of projects that require differing initial

investments.D. will never be greater than 1.0.

Internal rate of return56. According to the reinvestment rate assumption, which method of capital budgeting assumes

cash flows are reinvested at the project’s rate of return?A. payback period C. internal rate of returnB. net present value D. none of the above

62. The rate of interest that produces a zero net present value when a project’s discounted cash operating advantage is netted against its discounted net investment is the:A. Cost of capital C. Cutoff rateB. Discount rate D. Internal rate of return

57. A weakness of the internal rate of return method for screening investment projects is that it:A. Does not consider the time value of moneyB. Implicitly assumes that the company is able to reinvest cash flows from the project at the

company’s discount rateC. Implicitly assumes that the company is able to reinvest cash flows from the project at the

internal rate of returnD. Fails to consider the timing of cash flows

Comprehensive50. Which of the following methods of evaluating capital investment projects do not use a

percentage as a measurement unit?

486

Page 5: 11 x09 Capital Budgeting

Capital Budgeting

A. Payback period and net present valueB. Accounting rate of return and payback periodC. Net present value and internal rate of returnD. Internal rate of return and payback period

Relationships among NPV, PI & IRR24. If a company’s required rate of return is 12 percent and in using the profitability index method,

a project’s index is greater than 1.0, this indicates that the project’s rate of return isA. equal to 12 percent. C. less than 12 percent.B. greater than 12 percent. D. dependent on the size of the investment.

25. If the present value of the future cash flows for an investment equals the required investment, the IRR isA. equal to the cutoff rate.B. equal to the cost of borrowed capital.C. equal to zero.D. lower than the company’s cutoff rate return.

27. The relationship between payback period and IRR is thatA. a payback period of less than one-half the life of a project will yield an IRR lower than the

target rate.B. the payback period is the present value factor for the IRR.C. a project whose payback period does not meet the company’s cutoff rate for payback will

not meet the company’s criterion for IRR.D. none of the above.

67. When comparing NPV and IRR, which is not true?A. With NPV, the discount rate can be adjusted to take into account increased risk and the

uncertainty of cash flowsB. With IRR, cash flows can be adjusted to account for riskC. NPV can be used to compare investments of various size or magnitudeD. Both NPV and IRR can be used for screening decisions

Sensitivity analysis13. In capital budgeting, sensitivity analysis is used

A. to determine whether an investment is profitable.B. to see how a decision would be affected by changes in variables.C. to test the relationship of the IRR and NPV.D. to evaluate mutually exclusive investments.

15. An approach that uses a number of outcome estimates to get a sense of the variability among

potential returns isA. the discounted cash flow technique.B. the net present value method.C. risk analysis.D. sensitivity analysis.

42. Sensitivity analysis is the study of how the outcome of a decision making processA. changes as one or more of the assumptions changeB. remains the same even though one or more of the assumptions changeC. changes even though one or more of the assumptions do not changeD. does not change as the assumptions do not change either

64. Sensitivity analysis is:A. An appropriate response to uncertainty in cash flow projectionsB. Useful in measuring the variance of the Fisher rateC. Typically conducted in the post investment auditD. Useful to compare projects requiring vastly different levels of initial investment

IRR = 058. if the internal rate of return on an investment is zero:

A. its NPV is positive.B. its annual cash flows equal its required investment.C. it is generally a wise investment.D. its cash flows decrease over its life.

Change in NPV59. Which of the following would decrease the net present value of a project?

A. A decrease in the income tax rateB. A decrease in the initial investmentC. An increase in the useful life of the projectD. An increase in the discount rate

Effect of change in cost of capital26. All other things being equal, as cost of capital increases

A. more capital projects will probably be acceptable.B. fewer capital projects will probably be acceptable.C. the number of capital projects that are acceptable will change, but the direction of the

change is not determinable just by knowing the direction of the change in cost of capital.D. the company will probably want to borrow money rather than issue stock.

Effect of change in residual value

487

Page 6: 11 x09 Capital Budgeting

Capital Budgeting

23. Assuming that a project has already been evaluated using the following techniques, the evaluation under which technique is least likely to be affected by an increase in the estimated residual value of the project?A. Payback Period. C. Net Present Value.B. Internal Rate of Return. D. Profitability Index.

Decision rules – independent projects68. What type of decision involves deciding if an investment meets a predetermined standard?

A. Investment decisions C. Management decisionsB. Screening decisions D. Preference decisions

Payback period46. If a payback period for a project is greater than its expected useful life, the

A. project will always be profitable.B. entire initial investment will not be recovered.C. project would only be acceptable if the company’s cost of capital was low.D. project’s return will always exceed the company’s cost of capital.

Net present value61. An analysis of a proposal by the net present value method indicated that the present value of

future cash inflows exceeded the amount to be invested. Which of the following statements best describes the results of this analysis?A. The proposal is desirable and the rate of return expected from the proposal exceeds the

minimum rate used for the analysisB. The proposal is desirable and the rate of return expected from the proposal is less than

the minimum rate used for the analysisC. The proposal is undesirable and the rate of return expected from the proposal is less than

the minimum rate used for the analysisD. The proposal is undesirable and the rate of return expected from the proposal exceeds

the minimum rate used for the analysis

63. NPV indicates a project is deemed desirable (acceptable) when the NPV isA. greater than or equal to zeroB. less than zeroC. greater than or equal to the risk-adjusted cost of capitalD. less than or equal to the risk-adjusted cost of capital

Internal rate of return12. If Arbitrary Company wants to use IRR to evaluate long-term decisions and to establish a

cutoff rate of return, it must be sure that the cutoff rate isA. at least equal to its cost of capital.

B. at least equal to the rate used by similar companies.C. greater than the IRR on projects accepted in the past.D. greater than the current book rate of return.

NPV & IRR19. The NPV and IRR methods give

A. the same decision (accept or reject) for any single investment.B. the same choice from among mutually exclusive investments.C. different rankings of projects with unequal lives.D. the same rankings of projects with different required investments.

Decision rule – mutually exclusive projects71. Mutually exclusive projects are those that:

A. if accepted, preclude the acceptance of competing projects.B. if accepted, can have a negative effect on the company’s profit.C. if accepted, can also lead to the acceptance of a competing project.D. require all managers to consider.

28. In choosing from among mutually exclusive investments the manager should normally select the one with the highestA. Net present value. C. Profitability index.B. Internal rate return. D. Book rate of return.

53. Why do the NPV method and the IRR method sometimes produce different rankings of mutually exclusive investment projects?A. The NPV method does not assume reinvestment of cash flows while the IRR method

assumes the cash flows will be reinvested at the internal rate of return.B. The NPV method assumes a reinvestment rate equal to the discount rate while the IRR

method assumes a reinvestment rate equal to the internal rate of return.C. The IRR method does not assume reinvestment of the cash flows while the NPV assumes

the reinvestment rate is equal to the discount rate.D. The NPV method assumes a reinvestment rate equal to the bank loan interest rate while

the IRR method assumes a reinvestment rate equal to the discount rate.

Post-audit16. Post-audit of capital projects

A. is usually conclusive.B. is done using different evaluation techniques than were used in making the original capital

budgeting decision.C. provides a formal mechanism by which the company can determine whether existing

projects should be supported or terminated.

488

Page 7: 11 x09 Capital Budgeting

Capital Budgeting

D. all of the above.

17. A thorough evaluation of how well a project’s actual performance matches the projections made when the project was proposed is called aA. pre-audit. C. sensitivity analysis.B. post-audit. D. risk analysis.

37. A follow-up evaluation of a capital project is performed to see that investment expenditures are proceeding on time and on budget, to compare actual cash flows with those originally predicted, and to evaluate continuation of the project. This follow-up is called aA. postaudit. C. management auditB. performance evaluation D. project review

65. Companies use post audits to:A. chastise managers whose project does not exceed projections.B. prove to managers that they should have accepted projects they previously rejected.C. have the managers revise poorly performing projects so the projects will have larger

return in the future.D. provide feedback that enables managers to improve the accuracy of the projections of

future cash flows, thereby maximizing the quality of the firm’s capital investments.

PROBLEMS:Net Investment1. Bruell Company is considering to replace its old equipment with a new one. The old

equipment had a net book value of P100,000, 4 remaining useful life with P25,000 depreciation each year. The old equipment can be sold at P80,000. The new equipment costs P160,000, have a 4-year life. Cash savings on operating expenses before 40% taxes amount to P50,000 per year. What is the amount of investment in the new equipment?A. P160,000 C. P 80,000B. P 72,000 D. P 68,000

Operating Cash FlowCash Flow Before tax2. Taal Company is considering the purchase of a machine that promises to reduce operating

costs by equal amounts every year of its 6-year useful life. The machine will cost P840,000 and has no salvage value. The machine has a 20% internal rate of return. Taal Company is subject to 40% income tax rate. The present value of 1 for 6 periods at 20% is 3.326, and at the end of 6 periods is 0.3349. The approximate annual cash savings before tax is closest to:A. P252,555 C. P187,592

B. P112,555 D. P327,592

Increase in Annual Income Tax3. Mayon Company is considering replacing its old machine with a new and more efficient one.

The old machine has book value of P100,000, a remaining useful life of 4 years, and annual straight-line depreciation of P25,000. The existing machine has a current market value of P80,000. The replacement machine would cost P160,000, have a 4-year life, and will save P50,000 per year in cash operating costs. If the replacement machine would be depreciated using the straight-line method and the tax rate is 40%, what should be the increase in annual income taxes?A. P14,000 C. P40,000B. P28,000 D. P 4,000

Depreciation & Taxes4. Prime Consulting, Inc. operates consulting offices in Manila, Olongapo, and Cebu. The firm is

presently considering an investment in a new mainframe computer and communication software. The computer would cost P6 million and have an expected life of 8 years. For tax purposes, the computer can be depreciated using either straight-line method or Sum-of-the-Years’-Digits (SYD) method over five years. No salvage value is recognized in computing depreciation expense and no salvage value is expected at the end of the life of the equipment. The company’s cost of capital is 10 percent and its tax rate is 40 percent.The present value of annuity of 1 for 5 periods is 3.791 and for 8 periods is 5.335. The present values of 1 end of each period are:

1 0.9091 5 0.62092 0.8264 6 0.56453 0.6513 7 0.51324 0.6830 8 0.4665

The present value of the net advantage of using SYD method of depreciation with a five-year life instead of straight-line method of depreciating the equipment is:A. P 86,224 C. P215,560B. P115,168 D. P287,893

5. For P450,000, Maleen Corporation purchased a new machine with an estimated useful life of five years with no salvage value. The machine is expected to produce cash flow from operations, net of 40 percent income taxes, as follows:

First year P160,000Second year 140,000Third year 180,000Fourth year 120,000Fifth year 100,000

Maleen will use the sum-of-the-years-digits’ method to depreciate the new machine as follows:

489

Page 8: 11 x09 Capital Budgeting

Capital Budgeting

First year P150,000Second year 120,000Third year 90,000Fourth year 60,000Fifth year 30,000

The present value of 1 for 5 periods at 12 percent is 3.60478. The present values of 1 at 12 percent at end of each period are:

End of: Period 1 0.89280Period 2 0.79719Period 3 0.71178Period 4 0.63552Period 5 0.56743

Had Maleen used straight-line method of depreciation instead of declining method, what is the difference in net present value provided by the machine at a discount rate of 12 percent?A. Increase of P 9,750 C. Decrease of P24,376B. Decrease of P 9,750 D. Increase of P24,376

Accounting rate of returnBased on initial investment6. A piece of labor saving equipment that Marubeni Electronics Company could use to reduce

costs in one of its plants in Angeles City has just come onto the market. Relevant data relating to the equipment follow:

Purchase cost of the equipment P432,000Annual cost savings that will be provided by the equipment 90,000Life of the equipment 12 years

What is the simple rate of return to be provided by the equipment?A. Between 15% and 18%. C. 20.83%.B. 25.00%. D. 12.50%.

Based on average investment7. The BIBO Company has made an investment in video and recording equipment that costs

P106,700. The equipment is expected to generate cash inflows of P20,000 per year. How many years will the equipment have to be used to provide the company with a 10 percent average accounting rate of return on its investment?A. 7.28 years C. 9.05 yearsB. 5.55 years D. 4.75 years

8. Show Company is negotiating to purchase an equipment that would cost P200,000, with the expectation that P40,000 per year could be saved in after-tax cash operating costs if the equipment were acquired. The equipment’s estimated useful life is 10 years, with no salvage

value, and would be depreciated by the straight-line method. Show Company’s minimum desired rate of return is 12 percent. The present value of an annuity of 1 at 12 percent for 10 periods is 5.65. The present value of 1 due in 10 periods, at 12 percent, is 0.322.The average accrual accounting rate of return (ARR) during the first year of asset’s use is:A. 20.0 percent C. 10.0 percentB. 10.5 percent D. 40.0 percent

9. An asset was purchased for P66,000. The asset is expected to last for 6 years and will have a salvage value of P16,000. The company expects the income before tax to be P7,200 and the tax rate applicable to the company is 30%. What is the average return on investment (accounting rate of return)?A. 17.6% C. 10.9%B. 7.6% D. 12.3%

Net Investment10. The Makabayan Company is planning to purchase a new machine which it will depreciate, for

book purposes, on a straight-line basis over a ten-year period with no salvage value and a full

1 . Answer: BInitial amount of investment 160,000Less Cash inflow (decrease in outflow) at period 0: MV of old equipment 80,000 Tax benefits on loss on sales (20,000 x .4) 8,000 88,000Net investment 72,000

2 . Answer: DATCF = Net investment ÷ Payback periodATCF (840,000 ÷ 3.326) 252,555Net income (252,555 – 140,000) 112,555Before-tax income (112,555 ÷ 0.60) 187,592Before-tax savings (187,592 + 140,000) 327.592

The computation of after-tax cash flows, given the amount of investment and internal rate of return or PV of annuity of 1 discounted at IRR is the reverse of the computation of payback period. Remember that the payback method, though a nondiscounted technique, is closely related to internal rate of return because the payback period is exactly the present value of annuity of 1 if they are discounted using the internal rate of return.

3 . Answer: AAnnual savings on expenses P50,000Less: Additional depreciation (40,000 – 25,000) 15,000 Additional taxable income 35,000

490

Page 9: 11 x09 Capital Budgeting

Capital Budgeting

year’s depreciation taken in the year of acquisition. The new machine is expected to produce cash flows from operations, net of income taxes, of P66,000 a year in each of the next ten years. The accounting (book value) rate of return on the initial investment is expected to be 12 percent. How much will the new machine cost?A. P300,000 C. P550,000B. P660,000 D. P792,000

11. The Fields Company is planning to purchase a new machine which it will depreciate, for book purposes, on a straight-line basis over a ten-year period with no salvage value and a full year’s

Additional tax (35,000 x 40%) P14,000 Additional depreciation can be easily calculated by subtracting the book value of the old machine from the cost of new machine and then the difference divided by the useful life (160,000 – 100,000) ÷ 4 = 15,000.

4 10. Answer: BYearSYDStraight LineDifferencePresent Value12,000,0001,200,000800,000 727,28021,600,0001,200,000400,000330,56031,200,0001,200,000 -04

800,0001,200,000(400,000) (273,200)5 400,0001,200,000(800,000) (496,720)Total present value of difference in depreciation287,920Tax Rate40%Present value of net advantage115,168

6 . Answer: DAnnual cost savings 90,000Less depreciation (432,000 ÷ 12) 36,000Annual income 54,000Simple Rate of Return: 54,000 ÷ 432,000 12.5 %

7 . Answer: AThe useful life of the project can be calculated by using the computational pattern for Accounting Rate of Return:

Net investment 106,700Divide by Depreciation expense

CFAT 20,000Less: Net income (106,700 x 5%) 14,665 5,335

Average life (in years) 7.28* 10% ARR based on average investment = 5% ARR based on initial investment

8 . Answer: BARR = Average annual net income ÷ Average Investment Annual after-tax cash flow 40,000Less Depreciation 20,000Net Income 20,000

depreciation taken in the year of acquisition. The new machine is expected to produce cash flow from operations, net of income taxes, of P66,000 a year in each of the next ten years. The accounting (book value) rate of return on the initial investment is expected to be 12%. How much will the new machine cost?A. P300,000 C. P660,000B. P550,000 D. P792,000

CFAT12. The Hills Company, a calendar company, purchased a new machine for P280,000 on January

1. Depreciation for tax purposes will be P35,000 annually for eight years. The accounting (book value) rate of return (ARR) is expected to be 15% on the initial increase in required investment. On the assumption of a uniform cash inflow, this investment is expected to provide annual cash flow from operations, net of income taxes, ofA. P35,000 C. P42,000B. P40,250 D. P77,000

Payback Period13. If an asset costs P35,000 and is expected to have a P5,000 salvage value at the end of its ten-

year life, and generates annual net cash inflows of P5,000 each year, the cash payback period isA. 8 years C. 6 yearsB. 7 years D. 5 years

14. Consider a project that requires cash outflow of P50,000 with a life of eight years and a salvage value of P5,000. Annual before-tax cash inflow amounts to P10,000 assuming a tax rate of 30% and a required rate of return of 8%. Salvage value is ignored in computing depreciation. The project has a payback period ofA. 5.0 years C. 6.0 yearsB. 5.6 years D. 6.6 years

15. The following incomplete information is provided for an investment decision.

Year Cash FlowDiscount

Factor (10%)Discounted Cash Flows

Cumulative Cash Flows

0 P(450,000) 1.000 P(450,000) P(450,000)1 280,000 .909 254,5202 210,000 .826

Divide by Average Investment (200,000 + 180,000)/2 190,000ARR: 10.5%

The problem asked for the average accounting rate of return for the first year of asset’s life.

491

Page 10: 11 x09 Capital Budgeting

Capital Budgeting

3 140,000 .751Using break-even time (BET) analysis, when will the investment be recovered?A. In 2.73 years C. At the end of year 2B. Longer than three years D. In 2.21 years

16. Orlando Corporation is considering an investment in a new cheese-cutting machine to replace its existing cheese cutter. Information on the existing machine and the replacement machine follow:

Cost of the new machine P400,000Net annual savings in operating costs 90,000Salvage value now of the old machine 60,000Salvage value of the old machine in 8 years 0Salvage value of the new machine in 8 years 50,000Estimated life of the new machine 8 years

What is the expected payback period for the new machine?A. 4.44 years C. 2.67 yearsB. 8.50 years D. 3.78 years

17. For P4,500,000, Siniloan Corporation purchased a new machine with an estimated useful life of five years with no salvage value at its retirement. The machine is expected to produce cash flow from operations, net of income taxes, as follows:

First year P 900,000Second year 1,200,000Third year 1,500,000Fourth year 900,000Fifth year 800,000

Siniloan will use the sum-of-the-years-digits’ method to depreciate the new machine as follows:

First year P1,500,000Second year 1,200,000Third year 900,000Fourth year 600,000Fifth year 300,000

What is the payback period for the machine?A. 3 years C. 5 yearsB. 4 years D. 2 years

18. Paz Insurance Company’s management is considering an advertising program that would require an initial expenditure of P165,500 and bring in additional sales over the next five years. The cost of advertising is immediately recognized as expense. The projected additional sales revenue in Year 1 is P75,000, with associated expenses of P25,000. The additional sales

revenue and expenses from the advertising program are projected to increase by 10 percent each year. Paz Insurance Company’s tax rate is 40 percent.The payback period for the advertising program is A. 4.6 years C. 3.0 yearsB. 1.9 years D. 2.5 years

19. The Leisure Company is considering the purchase of electronic pinball machines to place in amusement houses. The machines would cost a total of P300,000, have an eight-year useful life, and have a total salvage value of P20,000. Based on experience with other equipment, the company estimates that annual revenues and expenses associated with the machines would be as follows:

Revenues form use P200,000Less operating expenses Commissions to amusement houses P100,000 Insurance 7,000 Depreciation 35,000 Maintenance 18,000 160,000Net income P 40,000

Ignoring the effect of income taxes, the payback period for the pinball machines would beA. 3.73 years C. 4.0 years B. 3.23 years D. 7.5 years

Net Present Value20. It is the start of the year and Agudelo Company plans to replace its old

grinding equipment. The following information are made available by the management:

Old NewEquipment cost P70,000 P120,000Current salvage value 14,000 -Salvage value, end of useful life 5,000 16,000Annual operating costs 44,000 32,000Accumulated depreciation 55,300 -Estimated useful life 10 years 10 years

The company is not subject to tax and its cost of capital is 12%. What is the present value of all the relevant cash flows at time zero? A. (P 54,000) C. (P106,000)B. (P120,000) D. (P124,700)

21. Consider a project that requires an initial cash outflow of P500,000 with a life of eight years and a salvage value of P20,000 upon its retirement. Annual cash inflow before tax amounts to P100,000 and a tax rate of 30 percent will be applicable. The required minimum rate of return

492

Page 11: 11 x09 Capital Budgeting

Capital Budgeting

for this type of investment is 8 percent. The present value of 1 and the annuity of 1, discounted at 8 percent for 8 periods are 0.54 and 5.747, respectively. Salvage value is ignored in computing depreciation. The net present value amounts toA. P 7,560 C. P 17,606B. P 10,050 D. P 20,050

22. Zap Manufacturing has an investment opportunity to embark on a project where yearly revenues for five years are to be P400,000 and operating costs of P104,800. The equipment costs P1 million, and straight-line depreciation will be used for book and tax purposes. No salvage value is expected at the end of the project’s life. The company has a 40 percent marginal tax rate and a 10 percent cost of capital. The equipment manufacturer has offered a delayed payment plan of P560,500 per year at the end of the first and second years. There will be no changes in working capital.The present value of annuity of 1 for 5 periods is 3.7908 at 10 percent. The present values of 1 end of each period at 10 percent are:

Period 1 0.9091Period 2 0.8264Period 3 0.7513Period 4 0.6830Period 5 0.6209

The net present value if the equipment were purchased isA. P (87,977) C. P 1,922B. P (25,310) D. P (61,094)

23. Paz Insurance Company’s management is considering an advertising program that would require an initial expenditure of P165,500 and bring in additional sales over the next five years. The cost of advertising is immediately recognized as expense. The projected additional sales revenue in Year 1 is P75,000, with associated expenses of P25,000. The additional sales revenue and expenses from the advertising program are projected to increase by 10 percent each year. Paz Insurance Company’s tax rate is 40 percent.

The present value of 1 at 10 percent, end of each period:Period Present value of 1

1. 0.909092. 0.826453. 0.751314. 0.683015. 0.62092

The net present value of the advertising program would beA. P 37,064 C. P 29,136B. P(37,064) D. P(29,136)

24. Mario Hernandez plans to buy a haymaker. It costs P175,000 and is expected to last for five years. He presently hires 6 workers at P10,000 per month for each of the three harvesting months each year. The equipment would eliminate the need for two workers. Hernandez uses straight-line depreciation and projects a salvage value of P25,000. His tax rate is 25% and opportunity cost of funds is 12.0%. The present value of 1discounted at 12 percent at the end of 5 periods is 0.56743 and the present value of an annuity of 1 for 5 periods is 3.60478. Which of the following is true?A. The present value of cash flows in year 5 is P22,710B. NPV is P28,436C. NPV is P15,250D. NPV is P14,186

25. Tabucol Aggregates, Inc. plans to replace one of its machines with a new efficient one. The old machine has a net book value of P120,000 with remaining economic life of 4 years. This old machine can be sold for P80,000. If the new machine were acquired, the cash operating expenses will be reduced from P240,000 to P160,000 for each of the four years, the expected economic life of the new machine. The new machine will cost Tabucol a cash payment to the dealer of P300,000. The company is subject to 32 percent tax and for this kind of investment, a marginal cost of capital of 9 percent. The present value of annuity of 1 and the present value of 1 for 4 periods using 9 percent are 3.23972 and 0.70843, respectively.The net present value to be provided by the replacement of the old machine isA. P28,493 C. P46,794B. P15,693 D. P59,594

26. Zambales Mines, Inc. is contemplating the purchase of equipment to exploit a mineral deposit that is located on land to which the company has mineral rights. An engineering and cost analysis has been made, and it is expected that the following cash flows would be associated with opening and operating a mine in the area.

Cost of new equipment and timbers 2,750,000Working capital required 1,000,000Net annual cash receipts* 1,200,000Cost to construct new road in three years 400,000Salvage value of equipment in 4 years 650,000

*Receipts from sales of ore, less out-of-pocket costs for salaries, utilities, insurance, etc.It is estimated that the mineral deposit would be exhausted after four years of mining. At that point, the working capital would be released for reinvestment elsewhere. The company’s discount rate is 20%.The net present value for the project is:A. P 454,620. C. P(561,553)B. P (79,303). D. P(204,688).

493

Page 12: 11 x09 Capital Budgeting

Capital Budgeting

With inflation27. By the end of December 31, 2005, Alay Foundation is considering the purchase of a copying

machine for P80,000. The expected annual cash savings are expected to be P32,000 in the next four years. At the end of the four years, the machine will be discarded without any salvage value. All the cash savings are stated in number of pesos at December 31, 2006. The company expected that the inflation rate is constantly 5 percent each year. Hence, the first year’s cash inflow was adjusted for 5 percent inflation. For simplicity, all cash inflows are assumed to be at year-end. The present value at 14 % of 1 for 4 periods is 2.91371. The present value of 1 at end of each period are:

Period 1 0.87719Period 2 0.76947Period 3 0.67497 Period 4 0.59208

Using the nominal rate of return of 14 percent, the net present value for this machine isA. P12,239 C. P13,419B. P19,670 D. P27,936

28. Perpetual Foundation, Inc., a nonprofit organization, has one of its activities, the production of cookies for its snack food store. Several years ago, Perpetual Foundation, Inc. purchased a special cookie-cutting machine. As of December 31, 2006, this machine will have been used for three years. Management is considering the purchase of a newer, more efficient machine. If purchased, the new machine would be acquired on December 31, 2006. Management expects to sell 300,000 dozen cookies in each of the next six years. The selling price of the cookies is expected to average P1.15 per dozen.Perpetual Foundation, Inc. has two options: continue to operate the old machine, or sell the old machine and purchase the new machine. No trade-in was offered by the seller of the new machine. The following information has been assembled to help management decide which option is more desirable.

Old Machine New MachineOriginal cost of machine at acquisition P80,000 P120,000Remaining useful life as of 12/31/06 6 years 6 yearsExpected annual cash operating expenses: Variable cost per dozen P0.38 P0.29 Total fixed costs P21,000 P 11,000Estimated cash value of machines: December 31, 2006 P40,000 P120,000 December 31, 2012 P 7,000 P 20,000

Assume all operating revenues and expenses occur at the end of the year.The net advantage in present value of the better alternative is:

A. Retain Old Machine, P61,675.B. Buy New Machine, P61,675.C. Retain Old Machine, P16,345.D. Buy New Machine, P16,345.

Profitability index29. The Pambansang Kamao Corporation has to replace its completely damaged boiler machine

with a new one. The old machine has a net book value of P100,000 with zero market value; therefore it will give a tax shield, based on 35% tax rate if replaced, by P35,000. The company has a 10 percent cost of capital. Understandably, the new machine, through a uniform decrease in cash operating costs, will give a positive net present value, because this machine will provide an internal rate of return of 12 percent.The present values at 10% and 12%, respectively, are:

10% 12%Annuity of 1, 6 periods 4.35526 4.111411 end of 6 periods 0.56447 0.50663

If the machine were to be depreciated using straight-line method for 6 years without any salvage value, the estimated profitability index is:A. 1.20B. 1.06C. 1.07D. Cannot be determined from the information

30. The Mejicano Company is planning to purchase a piece of equipment that will reduce annual cash expenses over its 5-year useful life by equal amounts. The company will depreciate the equipment using straight-line method of depreciation based on estimated life of 5 years without any salvage value. The company is subject to 40 percent tax. The marginal cost of capital for this acquisition is 11.055 percent. The management accountant calculated that the internal rate of return based on the estimated after-tax cash flows is 12.386 percent and a net present value of P10,000. The president, however, wants to know the profitability index before he finally decides.What is the profitability index for this investment?A. 1.011 C. 1.022B. 1.034 D. 1.044

Internal Rate of Return31. Diamond Company is planning to buy a coin-operated machine costing P400,000. For book

and tax purposes, this machine will be depreciated P80,000 each year for five years. Diamond estimates that this machine will yield an annual inflow, net of depreciation and income taxes, of P120,000. Diamond’s desired rate of return on its investments is 12%. At the following discount rates, the NPVs of the investment in this machine are:

494

Page 13: 11 x09 Capital Budgeting

Capital Budgeting

Discount Rate NPV12% +P3,25814% + 1,19716% - 70818% - 2,474

Diamond’s expected IRR on its investment in this machine isA. 3.25% C. 16.00%B. 12.00% D. 15.30%

Required investment32. Kipling Company has invested in a project that has an eight-year life. It is expected that the

annual cash inflow from the project will be P20,000. Assuming that the project has a internal rate of return of 12%, how much was the initial investment in the project if the present value of annuity of 1 for 8 periods is 4.968 and the present value of 1 is 0.404?A. P160,000 C. P 80,800B. P 99,360 D. P 64,640

33. Katol Company invested in a machine with a useful life of six years and no salvage value. The machine was depreciated using the straight-line method. It was expected to produce annual cash inflow from operations, net of income taxes, of P6,000. The present value of an ordinary annuity of P1 for six periods at 10% is 4.355. The present value of P1 for six periods at 10% is 0.564. Assuming that Katol used a time- adjusted rate of return of 10%, what was the amount of the original investment?A. P10,640 C. P22,750B. P29,510 D. P26,130

34. The Forest Company is planning to invest in a machine with a useful life of five years and no salvage value. The machine is expected to produce cash flow from operations, net of income taxes, of P20,000 in each of the five years. Forest’s expected rate of return is 10%. Information on present value and future amount factors is as follows:

P E R I O D1 2 3 4 5

Present value of P1 at 10% .909 .826 .751 .683 .621Present value of an annuity of P1 at 10% .909 1.736 2.487 3.170 3.791Future amount of P1 at 10% 1.100 1.210 1.331 1.464 1.611Future amount of an annuity of P1 at 10%

1.00 2.100 3.310 4.641 6.105

How much will the machine cost?A. P 32,220 C. P 75,820

B. P 62,100 D. P122,100

Required unit sales35. Paper Products Company is considering a new product that will sell for P100 and has a

variable cost of P60. Expected volume is 20,000 units. New equipment costing P1,500,000 and having a five-year useful life and no salvage value is needed, and will be depreciated using the straight-line method. The machine has fixed cash operating costs of P200,000 per year. The firm is in the 40 percent tax bracket and has cost of capital of 12 percent. The present value of 1, end of five periods is 0.56743; present value of annuity of 1 for 5 periods is 3.60478.How many units per year the firm must sell for the investment to earn 12 percent internal rate of return?A. 17,338 C. 9,838B. 28,897 D. 12,338

Required selling price36. Bughaw Products Company is considering a new product that will sell for P100 and has a

variable cost of P60. Expected sales volume is 20,000 units. New equipment costing P1,500,000 with a five-year useful life and no terminal salvage value is needed. The machine will be depreciated using the straight-line method. The machine has cash operating costs of P200,000 per year. The firm is in the 40 percent tax bracket and has cost of capital of 12 percent. The present value of 1, end of five periods is 0.56743; present value of annuity of 1 for 5 periods is 3.60478.Suppose the 20,000 estimated sales volume is sound, but the price is in doubt, what is the selling price (rounded to nearest peso) needed to earn a 12 percent internal rate of return?A. P81.00 C. P70.00B. P95.00 D. P90.00

Required CFBT37. Aloha Co. is considering the purchase of a new ocean-going vessel that could potentially

reduce labor costs of its operation by a considerable margin. The new ship would cost P500,000 and would be fully depreciated by the straight-line method over 10 years. At the end of 10 years, the ship will have no value and will be sunk in some already polluted harbor. The Aloha Co.’s cost of capital is 12 percent, and its marginal tax rate is 40 percent. If the ship produces equal annual labor cost savings over its 10-year life, how much do the annual savings in labor costs need to be to generate a net present value of P0 on the project?Use the following PV: annuity of 1, 10 periods at 12% - 5.6502; end of 10th period – 0.32197.A. P 68,492 C. P114,154B. P147,487 D. P 88,492

Required CFAT

495

Page 14: 11 x09 Capital Budgeting

Capital Budgeting

38. Prudu Company has decided to invest in some new equipment. The equipment will have a three-year life and will produce a uniform series of cash savings. The net present value of the equipment is P1,750, using a discount rate of 8 percent. The internal rate of return is 12 percent.Present values at 8% and 12% respectively:

8%: Annuity – 2.5771; end of 3 periods, 0.793812%: Annuity – 2,4018; end of 3 periods, 0.7118

What is the amount of annual cash inflow?A. P 9,980 C. P23,240B. P21,342 D. P12,351

39. An asset is purchased for P120,000. It is expected to provide an additional P28,000 of annual net cash inflows. The asset has a 10-year life and an expected salvage value of P12,000. The hurdle rate is 10%. The present value of an annuity factor of 10% for 10 years is 6.1446, and the present value of P1, discounted for 10 years at 10% is 0.3855. Given the data provided, the minimum amount of annual cash inflows that would provide the 10% time-adjusted return is approximatelyA. P18,776 C. P24,400B. P26,600 D. P22,535

Required Increase in CFAT40. The following data pertain to Julian Corp. whose management is planning to purchase a unit of

equipment.1. Economic life of equipment – 8 years.2. Disposal value after 8 years – Zero.3. Estimated net annual cash inflows for each of the 8 years – P81,000.4. Time-adjusted internal rate of return – 14%5. Cost of capital of Bayan Muna – 16%6. The table of present values of P1 received annually for 8 years has these factors: at

14% = 4.639, at 16% = 4.3447. Depreciation is approximately P46,970 annually.

Find the required increase in annual cash inflows in order to have the time-adjusted rate of return approximately equal the cost of capital. A. P6,501 C. P4,344B. P5,501 D. P5,871

Required CFAT for a certain year41. A company is considering putting up P50,000 in a three-year project. The company’s

expected rate of return is 12%. The present value of P1.00 at 12% for one year is 0.893, for two years is 0.797, and for three years is 0.712. The cash flow, net of income taxes will be P18,000 (present value of P16,074) for the first year and P22,000 (present value of P17,534)

for the second year. Assuming that the rate of return is exactly 12%, the cash flow, net of income taxes, for the third year would beA. P23,022 C. P10,000B. P 7,120 D. P16,392

Required salvage value42. The Caravan Company is contemplating to purchase a machine that costs P800,000. The

machine is expected to last for 5 years with a salvage value of P50,000 at the end of the fifth year. If the machine were purchased, before-tax annual cash savings on operating expenses will be realized. Caravan Company will depreciate the machine using straight-line depreciation for 5 years, with the salvage value considered in the computation.The company has a 12 percent cost of capital and is subject to 40 percent tax rate.The present values using 12 percent are:

Annuity of 1 for 5 periods 3.60478Present value of 1, end of 5 periods 0.56743

The initial analysis indicated a net present value of P7,003. You believe the estimated before-tax cash savings are fairly determined but you are in doubt of the expected salvage value of the machine. How much is the estimated salvage value required if the investment has to yield an IRR of 12 percent?A. P41,800 C. P25,100B. P24,900 D. P44,600

Required value of intangible benefits43. Solidum Company is investigating the purchase of a piece of automated equipment that will

save P100,000 each year in direct labor and inventory carrying costs. This equipment costs P750,000 and is expected to have a 10-year useful life with no salvage value. The company requires a minimum 15% return on all equipment purchases. Management anticipates that this equipment will provide intangible benefits such as greater flexibility and higher quality output.

The PV of annuity of 1, 15% for 10 periods 5.01877The PV of 1, end 10 period 0.24718

What peso value per year would these intangible benefits have to have in order to make the equipment an acceptable investment?A. P248,123 C. P 61,331B. P 49,440 D. P 55,000

44. Altas, Inc., is considering investing in automated equipment with a ten-year useful life. Managers at Altas have estimated the cash flows associated with the tangible costs and benefits of automation, but have been unable to estimate the cash flows associated with the intangible benefits. Using the company’s 10% discount rate, the net present value of the cash flows associated with just the tangible costs and benefits is a negative P184,350. The

496

Page 15: 11 x09 Capital Budgeting

Capital Budgeting

present value of annuity of 1 at 10 percent for ten years is 6.145 while the present value of 1 is 0.386. How large would the annual net cash inflows from the intangible benefits have to be to make this a financially acceptable investment?A. P18,435. C. P35,000.B. P30,000. D. P37,236.

Indifference Point45. Moon Company uses a 10% discount rate and the total cost approach to capital budgeting

analysis. Both alternatives are Akda Investments which has a marginal cost of capital of 12 percent is evaluating two mutually exclusive projects (X and Y), which have the following projections:

PROJECT X PROJECT YInvestment P48,000 P83,225After-tax cash inflow 12,000 15,200Asset life 6 years 10 years

The indifference point for the two projects isA. 12.64% C. 12.00%B. 16.01% D. 19.33%

46. Silky Products is considering two pieces of machinery. The first machine costs P50,000 more than the second machine. During the two-year life of these two alternatives, the first machine has a P155,000 more cash flow in year one and a P110,000 less cash flow in year two than the seconds machine. All cash flows occur at year-end. The present value of 1 at 15 percent end of 1 period and 2 periods are 0.86957 and, 0.75614, respectively. The present value of 1 at 8 percent end of period 1 is 0.92593, and Period 2 is 0.85734.At what discount rate would Machine 1 be equally acceptable as machine 2’s?A. 9% C. 11%B. 10% D. 12%

Decision Rule – Independent Projects47. Sylvia Products is considering two types of machinery. The first machine costs P50,000 more

than the second machine. During the two-year life of these two alternatives, the first machine has a P155,000 more cash flow in year one and a P110,000 less cash flow in year two than the seconds machine. All cash flows occur at year-end. The present value of 1 at 15 percent end of 1 period and 2 periods are 0.86957 and, 0.75614, respectively. The present value of 1 at 8 percent end of period 1 is 0.92593, and Period 2 is 0.85734.Which machine should be purchased if the relevant discount rates are 15 percent and 8 percent, respectively?

15% Discount 8% DiscountA. Machine 1 Machine 1B. Machine 2 Machine 2

C. Machine 1 Machine 2D. Machine 2 Machine 1

ComprehensivePayback, NPV, ARRQuestion Nos. 71 through 73 are based on the following:Cayco Medical Center is considering purchasing an ultrasound machine for P950,000. The machine has a 10 – year life and an estimated salvage value of P55,000. Installation costs and freight charges will be P24,200 and P800, respectively. Newman uses straight-line depreciation.

The medical center estimates that the machine will be used five times a week with the average charges to the patient for ultrasound of P800. There are P10 in medical supplies and P40 of technician costs for each procedure performed using the machine. The present value of an annuity of 1 for 10 years at 9% is 6.418 while the present value of 1 for 10 years at 9% is 0.42241

48. The cash payback period is:A. 3.0 years C. 5.0 yearsB. 4.5 years D. 6.0 years

49. The project is expected to generate net present value of:A. P276,510 C. P331,510B. P299,743 D. P253,277

50. What is the accounting rate of return provided by the project?A. 20.0 percent C. 11.2 percentB. 10.6 percent D. 38.0 percent

NPV, CFAT, Maximum lost unit salesQuestion Nos. 75 through 77 are based on the following:Kabalikat Company has the opportunity to introduce a new product. Kabalikat expects the product to sell for P75 with variable cost per unit of P50. The annual fixed costs, excluding the amount of depreciation is P4,500,000. The company expects to sell 300,000 units. To produce the new product line, the company needs to purchase a new machine that costs P6,000,000. The new machine is expected to last for four years with a very negligible salvage value. The company has a policy of depreciating its machine for both book and tax purposes for four years. The company has a marginal cost of capital of 13.75 percent and is subject to tax rate of 40 percent.

51. The amount of annual after-tax cash flows is:A. P2,400,000 C. P 900,000B. P3,000,000 D. P1,500,000

497

Page 16: 11 x09 Capital Budgeting

Capital Budgeting

52. The machine’s net present value is:A. P2,786,100 C. P1,028,900B. P 928,500 D. P 150,270

53. Assuming that some of the 300,000 units that are expected as sales would be to group of customers who currently buy K-Z, another product of Kabalikat Company. This Product K-Z sells for P35 with variable cost of P20. How many units of K-Z can Kabalikat afford to lose before the purchase of the new machine becomes unattractive?A. 39,000 units C. 16,714 unitsB. 23,400 units D. 10,029 units

ARR, NPV, PI, PaybackQuestions 1 through 4 will be based on the following data:The management of Arleen Corporation is considering the purchase of a new machine costing P400,000. The company’s desired rate of return is 10%. The present value of P1 at compound interest of 10% for 1 through 5 years are 0.909, 0.826, 0.751, 0.683, and 0.621, respectively, and the present value of annuity of 1 for 5 periods at 10 percent is 3.79. In addition to the foregoing information, use the following data in determining the acceptability in this situation:

Year Income from Operations Net Cash Flow1 P100,000 P180,0002 40,000 120,0003 20,000 100,0004 10,000 90,000

5 10,000 90,000

54. The average rate of return for this investment is:A. 18 percent C. 58 percentB. 6 percent D. 10 percent

55. The net present value for this investment is:A. Positive P 36,400 C. Negative P 99,600B. Positive P 55,200 D. Negative P126,800

56. The present value index for this investment is:A. 0.88 C. 1.14B. 1.45 D. 0.70

57. The cash payback period for this investment is:A. 4 years C. 20 yearsB. 5 years D. 3 years

Payback, NPV, ARR, IRRUse the following information for questions 67 - 70Pillo Company is considering two capital investment proposals.

Estimates regarding each project are provided below:

Project MA Project PAInitial investment P2000,000 P300,000Annual net income 10,000 21,000Net annual cash inflow 50,000 71,000Estimated useful life 5 years 6 yearsSalvage value -0- -0-

The company requires a 10% rate of return on all new investments.

Present Value of an Annuity of 1

Period 9% 10% 11% 12%

498

Page 17: 11 x09 Capital Budgeting

Capital Budgeting

5 3.890 3.791 3.696 3.6056 4.486 4.355 4.231 4.111

58. The cash payback period for Project MA isA. 20 years C. 5 yearsB. 10 years D. 4 years

59. The net present value for Project PA isA. P309,204 C. P 50,000B. P 91,456 D. P 9,205

60. The annual rate of return for Project MA isA. 5% C. 25%B. 10% D. 50%

61. The internal rate of return for Project PA is closest toA. 10% C. 12%B. 11% D. none of these

Depreciation tax shield, CCFAT, Payback, NPV, IRRQuestion Nos. 86 through 90 are based on the following:Consider a project that requires cash outflow of P50,000 with a life of eight years and a salvage value of P2,000. Annual cash inflow amounts to P10,000 assuming a tax rate of 30% and a required rate of return of 8%. Salvage value is ignored in computing depreciation.

62. Annual depreciation tax shield amounts toA. P1,875 C. P8,875B. P7,000 D. P10,000

63. Annual cash flow after tax amounts toA. P 1,875 C. P 8,875B. P 7,000 D. P10,000

64. Payback amounts toA. 5.0 years C. 6.0 yearsB. 5.6 years D. 6.6 years

65. Net present value amounts toA. P 756 C. P1,756B. P1,005 D. P2,005

66. Internal rate of return on this project is approximatelA. 8.0% C. 9.0%B. 8.5% D. 9.5%

CFAT, NPV, IRRQuestions 46 rough 51 are based on the following:Home’s Pizza’s, Inc., operates pizza shops in several cities. One of the company’s most profitable shops is located adjacent to the large CPA review center in Manila. A small bakery next to the shop has just gone out of business, and Home’s Pizzas has an opportunity to lease the vacated space for P18,000 per year under a 15-year lease. Home’s management is considering two ways in which the available space might be used.

Alternative 1. The pizza shop in this location is currently selling 40,000 pizzas per year. Management is confident that sales could be increased by 75% by taking out the wall between the pizza shop and the vacant space and expanding the pizza outlet. Costs for remodeling and for new equipment would be P550,000. Management estimates that 20% of the new sales would be small pizzas, 50% would be medium pizzas, and 30% would be large pizzas. Selling prices and costs for ingredients for the three sizes of pizzas follow (per pizza):

Selling Price Cost of IngredientsSmall P 6.70 P1.30Medium 8.90 2.40Large 11.00 3.10

An additional P7,500 of working capital would be needed to carry the larger volume of business. This working capital would be released at the end of the lease term. The equipment would have a salvage value of P30,000 in 15 years, when the lease ends.Alternative 2. Home’s sales manager feels that the company needs to diversify its operations. He has suggested that an opening be cut in the wall between the pizza shop and the vacant space and that video games be placed in the space, along with a small snack bar. Costs for remodeling and for the snack bar facilities would be P290,000. The games would be leased from a large distributor of such equipment. The distributor has stated that based on the use of game centers elsewhere, Home’s could expect about 26,000 people to use the center each year and to spend an average of P5 each on the machines. In addition, it is estimated that the snack bar would provide a net cash inflow of P15,000 per year. An investment of P4,000 in working capital would be needed. This working capital investment would be released at the end of the lease term. The snack bar equipment would have a salvage value of about P12,000 in 15 years.

Home’s management is unsure which alternative to select and has asked you to help in making the decision. You have gathered the following information relating to added costs that would be incurred each year under the two alternatives:

499

Page 18: 11 x09 Capital Budgeting

Capital Budgeting

Expand the Pizza Shop Install the Game CenterRent- building space P18,000 P18,000Rent- video games --- 30,000Salaries 54,000 17,000Utilities 13,200 5,400Insurance and other 7,800 9,600

The company is currently using a 16 percent minimum acceptable rate of return for its capital investment. The present value of annuity of 1 at 16 percent for 15 periods is 5.575 and end of 15 periods is 0.108. The company is not liable to pay income taxes.

67. The incremental expected annual cash inflows from Alternative 1 is:A. P 90,000 C. P100,200B. P108,000 D. P201,000

68. The incremental expected annual cash inflows from Alternative 2 is:A. P 17,000 C. P 59,600B. P 65,000 D. P145,000

69. The net present value for Alternative 1 is:A. P48,650 C. P45,000B. P47,840 D. P32,500.

70. The net present value for Alternative 2 is:A. P21,021 C. P68,375B. P70,103 D. P12,807

71. Assume that the company decides to accept alternative 2. At the end of the first year, the company finds that only 21,000 people used the game center during the year (each person spent P5 on games). Also, the snack bar provided a net cash inflow of only P13,000. In light of this information, what is the net present value for alternative 2?A. P(80,422) C. P(82,150)B. P(76,422) D. P(80,854)

72. The sales manager has suggested that an advertising program be initiated to draw another 5,000 people into the game center each year. Assuming that another 5,000 people can be attracted into the center and that the snack bar receipts increase to the level originally estimated, how much can be spent on advertising each year and still allow the game center to provide a 16% rate of return?A. P70,103.00 C. P58,953.00

B. P 4,673.53 D. P12,574.53

Net Income, CFBT, ARR, Payback PeriodQuestions 52 through 56 are based on the following information:Pinewood Craft Company is considering the purchase of two different items of equipment, as described below:

Machine A. A compacting machine has just come onto the market that would permit Pinewood Craft Company to compress sawdust into various shelving products. At present the sawdust is disposed of as a waste product. The following information is available on the machine:

a. The machine would cost P420,000 and would have a 10% salvage value at the end of its 12-year useful life. The company uses straight-line depreciation and considers salvage value in computing depreciation deductions.

b. The shelving products manufactured from use of the machine would generate revenues of P300,000 per year. Variable manufacturing costs would be 20% of sales.

c. Fixed expenses associated with the new shelving products would be (per year): advertising, P40,000; salaries, P110,000; utilities, P5,200; and insurance, P800.

Machine B. A second machine has come onto the market that would allow Pinewood Craft Company to automate a sanding process that is now done largely by hand. The following information is available:

a. The new sanding machine would cost P234,000 and would have no salvage value at the end of its 13-year useful life. The company would use straight-line depreciation on the new machine.

b. Several old pieces of sanding equipment that are fully depreciated would be disposed of at a scrap value of P9,000.

c. The new sanding machine would provide substantial annual savings in cash operating costs. It would require an operator at an annual salary of P16,350 and P5,400 in annual maintenance costs. The current, hand-operated sanding procedure costs the company P78,000 per year in total.

Pinewood Craft Company requires a simple rate of return of 15% on all equipment purchases. Also, the company will not purchase equipment unless the equipment has a payback period of 4.0 years or less. (In all the following questions, please ignore income tax effect)

73. The expected income each year from the new shelving products (Machine A) is:A. P 52,500 C. P 84,000B. P240,000 D. P 92,500

500

Page 19: 11 x09 Capital Budgeting

Capital Budgeting

74. The annual savings in cost if Machine B is purchased isA. P56,250 C. P38,250B. P43,250 D. P21,750

75. The simple rate (%) of return for Machine A is:A. 12.5 percent C. 25.0 percentB. 20.0 percent D. 18.0 percent

76. The simple rate of return for Machine B is:A. 16.3 percent C. 25.0 percentB. 17.0 percent D. 34.0 percent

77. The payback period for Machine A is:A. 3.0 years C. 5.0 years B. 4.5 years D. 7.5 years

78. The payback period for Machine B is:A. 4.0 years. C. 6.1 years.B. 4.2 years. D. 5.9 years.

Net Investment, CFBT, Tax Benefits, NPV, Depreciation Tax Shield, Question Nos. 58 through 63 are based on the following:Turkey Company’s average production of valve stems over the past three years has been 80,000 units each year. Expectations are that this volume will remain constant over the next four years. Cost records indicate that unit product costs for the valve stem over the last several years have been as follows:

Direct materials P 3.60Direct labor 3.90Variable manufacturing overhead 1.50Fixed manufacturing overhead* 9.00Unit product cost P18.00

*Depreciation of tools (that must now be replaced) accounts for one-third of the fixed overhead. The balance is for other fixed overhead costs of the factor that require cash expenditures.

If the specialized tools are purchased, they will cost P2,500,000 and will have a disposal value of P100,000 at the end of their four-year useful life. Turkey Company has a 30% tax rate, and management requires a 12% after-tax return on investment. Straight-line depreciation would be used for financial reporting purposes, but for the tax purposes, the following variable depreciation

each year will be used.

Year 1 P 832,500Year 2 1,112,500Year 3 370,000Year 4 185,000

The sales representative for the manufacturer of the specialized tools has stated, “The new tools will allow direct labor and variable overhead to be reduced by P1.60 per unit.” Data from another company using identical tools and experiencing similar operating conditions, except that annual production generally averages 100,000 units, confirms the direct labor and variable overhead cost savings. However, the other company indicates that it experienced an increase in raw material cost due to the higher quality of material that had to be used with the new tools. The other company indicates that its unit product costs have been as follows:

Direct materials P 4.50Direct labor 3.00Variable manufacturing overhead 0.80Fixed manufacturing overhead 10.80Unit product cost P19.10

Referring to the figures above, the production manager stated, “These numbers look great until you consider the difference in volume. Even with the reduction in labor and variable overhead cost, I’ll bet our total unit cost figure would increase to over P20 with the new tools.”

Although the old tools being used by Turkey Company are now fully depreciated, they have a salvage value of P45,000. These tools will be sold if the new tools are purchased; however if the new tools are not purchased, then the old tools will be retained as standby equipment. Turkey Company’s accounting department has confirmed that total fixed manufacturing overhead costs, other than depreciation, will not change regardless of the decision made concerning the valve stems. However, the accounting department has estimated that working capital needs will increase by P60,000 if the new tools are purchased due to the higher quality of material required in the manufacture of the valve stems.

The present values of 1 at the end of each period using 12 percent are:Period 1 0.89286Period 2 0.79719Period 3 0.71178Period 4 0.63552PV of annuity of 1, 4 periods 3.03735

501

Page 20: 11 x09 Capital Budgeting

Capital Budgeting

79. The net investment in new tools amounted to:A. P1,873,300. C. P2,528,500.B. P2,515,000. D. P2,546.500.

80. How much annual cost savings will be generated if the Turkey Company purchases the new tools?A. P 128,000 C. P 936,000B. P 216,000 D. P1,008,000

81. The present value of tax benefits expected from the use of the new machine tools is:A. P 603,333 C. P1,407,777B. P 804,444 D. P2,011,111

82. The present value of the salvage value of the new tools to be received at the end of fourth year isA. P 63,552. C. P 44,486.B. P 19,065. D. P212,615.

83. Using the minimum acceptable rate of return of 12 percent, the net present value of the investment in new tools isA. P108,913. C. P147,073.B. P127,979. D. P166,139.

84. The net advantage of the use of declining method of depreciation instead of straight-line method isA. P 33,830. C. P112,767.B. P 56,610. D. P147,731.

Net Investment, CFAT, Depreciation tax shield, NPVQuestion Nos. 77 through 82 are based on the following:Franzen Company manufactures three different models of paper shredders including the waste container, which serves as the base. While the shredder heads are different for all three models, the waste container is the same. The number of waste containers that Franzen will need during the next five years is estimated as follows:

2007 50,0002008 50,0002009 52,0002010 55,0002011 55,000

The equipment used to manufacture the waste container must be replaced because it is broken and cannot be repaired. The new equipment would have a purchase price of P945,000 with terms 2/10, n/30; the company’s policy is to take all purchase discounts. The freight on the equipment would be P11,000, and installation costs would total P22,900. The equipment would be purchased in December 2006 and placed into service on January 1, 2007. It would have a five-year economic life and would have the following depreciation. The equipment is expected to have a salvage value of P12,000 at the end of its economic life in 2011. The new equipment would be more efficient than the old equipment, resulting in a 25 percent reduction in both direct material and variable overhead. The savings in direct material would result in an additional one-time decrease in working capital requirements of P2,500, resulting from a reduction in direct material inventories. This working capital reduction would be recognized at the time of equipment acquisition.

The old equipment is fully depreciated and is not included in the fixed overhead. The old equipment from the plant can be sold for a salvage amount of P1,500. Rather than replace the equipment, one of Franzen’s production managers has suggested that the waste containers be purchased. One supplier has quoted a price of P27 per container. This price is P8 less than Franzen’s current manufacturing cost, which is presented below.

Direct materials P10Direct labor 8Variable overhead 6Fixed overhead: Supervision P2 Facilities 5 General 4 11Total unit cost P35

Franzen uses a plantwide fixed overhead rate in its operations. If the waste containers are purchase outside, the salary and benfits of one supervisor, included in fixed overhead of P45,000 would be eliminated. There would be no other changes in the other cash and noncash items included in fixed overhead except depreciation on the new equipment.

The new equipment will be depreciated according to the following declining amounts:Year Depreciation2007 P319,9682008 426,7202009 142,1762010 71,1362011 0

Franzen is subject to a 40 percent tax rate. Management assumes that all cash flows occur at the

502

Page 21: 11 x09 Capital Budgeting

Capital Budgeting

end of the year and uses a 12 percent after-tax discount rate.

85. The initial net cash outflows if the company decides to continue making the waste containers is:A. P 956,600 C. P 978,900B. P 975,500 D. P1,455,613

86. The total after-tax cash outflows, excluding the initial cash outflows, if the new equipment is purchased are:A. P 956,600 C. P2,918,300B. P2,887,800 (defective) D. P3,279,000

87. The present value of the total depreciation shield is:A. P308,920 C. P307,826B. P313,500 D. P321,303

88. The total relevant after-tax costs to buy the waste containers are:A. P2,829,240 C. P4,243,500 (defectiveB. P3,039,662 D. P7,074,000

89. What is the net present value of the purchase alternative?A. P3,039,662 (defective) C. P2,083,062B. P2,730,742 D. P2,718,359

90. What is the net present value of the make alternative?A. P2,036,603 C. P2,996,603B. P3,039,662 D. P2,993,203 (defective)

ANSWER EXPLANATIONS

503

Page 22: 11 x09 Capital Budgeting

Capital Budgeting

5 . Answer: BSYDSLDifferencePresent Value1 150,00090,00060,00053,5682 120,00090,00030,00023,9163 90,00090,000-04 60,00090,000(30,000)(19,066)5 30,00090,000(60,000) (34,046)Total of present values of depreciation24,372Tax rate40%Present value of net advantage 9,749SYD method provides a higher present value on tax benefits because of less amount of tax during year 1 & 2. In year 4 and 5, the use of SYD requires higher taxes but their equivalent present values are lower already.

504

Page 23: 11 x09 Capital Budgeting

Capital Budgeting

9 . Answer: DThe average (accounting) rate of return is determined by dividing the annual after-tax net income by the average cost of the investment, (beginning book value + ending book value)/2.

After tax income (P7,200 - (P7,200 x 30%)) P 5,040Average investment: (P66,000 + 16,000) ÷ 2 P41,000Accounting rate of return: P5,040/P41,000) 12.3%

10 . Answer: A (ATCF – Depreciation) ÷ Initial investment = Accounting Rate of ReturnLet X = Initial investment(66,000 – 0.10X) ÷ X = 0.12 66,000 - .10X = .12X

.22X = 66,000 X = 300,000

505

Page 24: 11 x09 Capital Budgeting

Capital Budgeting

11 . Answer: ANet Income: = 66,000 - .10XAAR = NI/ Investment.12 = (66,000 - .10X) / X.12X = 66,000 - .10X.22 X = 66,000 X = 300,000

506

Page 25: 11 x09 Capital Budgeting

Capital Budgeting

12 . Answer: DNet Income (280,000 x 15%) 42,000

Add back depreciation 35,000ATCF 77,000

507

Page 26: 11 x09 Capital Budgeting

Capital Budgeting

13 . Answer: BPayback period = Initial amount of investment ÷ Annual after-tax cash flows

P35,000 ÷ P5,000 = 7 years

14 . Answer: BNet investment 50,000Divide by CFAT (10,000 x 0.7) ÷ (50,000 ÷ 8 x 0.3) 8,875Payback period 5.6 years

15 . Answer: DCumulative cash flows end of Year 1 (450,000) – 254,520 (195,480)Discounted cash flow for Year 2 173,460Cumulative cash flows, end of Year 2 ( 22,020)Break-even time 2 + (22,020 ÷ 105,140) 2.21 years

16 . Answer: DCost of the new machine 400,000Salvage value of old machine at period zero 60,000Net investment (Outflows) 340,000Divide by cash flow after tax 90,000Payback period 3.78 years

17 . Answer: B Cash InflowUnrecovered OutflowOutflows(4,500,000)First year900,000(3,600,000)Second year1,200,000(2,400,000)Third year1,500,000( 900,000)Fourth year 900,0000

Payback Period: At the end of 4 periods, the initial outflows are fully recovered.Note to the CPA Candidates: A modified question for this problem is to compute the Present Value of the net advantage of using sum-of-the-years’ digits of depreciation instead of straight-line method.

18 . Answer: CCash inflowsInvestmentPeriod 0(99,300)Period 1 (75,000 – 25,000) x .6 30,000(69,300)Period 2 ( 30,000 x 1.10) 33,000(36,300)Period 3 (33,000 x 1.10) 36,300 -0-At the end of the third

year, investment is fully recovered.The net investment of 99,300 is net of tax benefit, (165,500 x .6)

508

Page 27: 11 x09 Capital Budgeting

Capital Budgeting

19 . Answer: CBefore-tax cash flow = 40,000 + 35,000 75,000Payback period: 300,000 ÷ 75,000 4 years

20 . Answer: CThere are two cash flows at time zero: P120,000 outflow and P14,000 inflow.Net cash outflow (120,000 – 14,000) = 106,000

21 . Answer: CComputation of Cash Flow After-tax

CFBT 100,000 x 0.7 70,000Depreciation tax shield 62,500 x 0.3 18,750CFAT 88,750

Computation of Net Present Value:PV of ATCF: 88,750 x 5.747 510,046PV of After-tax Salvage Value: 20,000 x 0.70 x 0.54 7,560Total 517,606Investment 500,000Net Present Value 17,606

The problem assumed that the salvage value is ignored in the computation of annual depreciation so that the annual cash flows will be greater. The problem did not include among the choices the assumption that salvage value will be deducted from the cost in computing the amount of annual depreciation.

22 . Answer: BAnnual revenues 400,000Less cash operating costs 104,800Cash flow before tax 295,200Less Depreciation (1M ÷ 5) 200,000Income before tax 95,200Less income tax (40%) 28,080Net income 57,120Add back depreciation 200,000

509

Page 28: 11 x09 Capital Budgeting

Capital Budgeting

ATCF 257,120PV of ATCF, n=5; k=10% 257,120 x 3.7908 974,690

23 . Answer: APresent value of cash returns: (30,000 x 0.90909) x 5 periods 136,364Net investment 99,300Net present value 37,064

Note: Because the constant growth rate and the discount rate are both 10%, the present value for each period is constant.

24 . Answer: BSavings (2 workers, each P10,000 for 3 months) 2 x P10,000 x 3 P60,000Depreciation (175,000 – 25,000) ÷ 5 years P30,000After-tax cash savings: (60,000 x 0.75) + (30,000 x 0.25) P52,500Present value of after-tax cash savings (52,500 x 3.60478) P189,250Present value of Salvage Value (25,000 x 0.56743) 14,186Total 203,436Investment 175,000Net Present Value P 28,436

25 . Answer: BComputation of net investment:

Cash purchase price 300,000Less: MV of old machine 80,000 Tax shield on loss on sale (40,000 x 0.32) 12,800 92,800Net investment 207,200

Annual cash savings before tax (240,000 – 160,000) 80,000Additional depreciation (300,000 – 120,000) ÷ 4 45,000Additional taxable income 35,000Less Additional tax (35,000 x 0.32) 11,200Net income 23,800Add back depreciation 45,000After-tax cash flow 68,800Alternative computation for ATCF:

510

Page 29: 11 x09 Capital Budgeting

Capital Budgeting

Investment 1,000,00026 . Answer: BPV of annual cash receipts 1,200,000 x 2.58872 3,106,463PV of salvage value 650,000 x 0.48225 313,462PV of return of working capital 1,000,000 x 0.48225 482,250Cost of new equipment and timbers (2,750,000)Working capital (1,000,000)PV of cost of construction of road 400,000 x .5787 ( 231,480)Negative net present value (79,303)

27 . Answer: BPeriodNominal Cash SavingsPV FactorPresent Value132,000 0.8779028,070.08232,000 x 1.0533,600 0.7694725,854.19332,000 x 1.05235,280 0.6749723,812.94432,000 x 1.05337,044

0.5920821,933.01Total99,670.22Investment80,000.00NPV19,670.22Note that all the annual cash inflows are adjusted by one period.

28 . Answer: BThe solution used total analysis approach in computing present value.Retain the Old Machine:Present value of annual cash outlay CFAT (300,000 x P0.38) + P21,000 = P135,000 PVCFAT (135,000 x 3.6847) P497,435Present value of salvage value (7,000 x 0.41044) ( 2,873) Total P494,562

Buy New machine:Present Value of Annual cash outlay CFAT (300,000 x P0.29) + P11,000 = P98,000 PVCFAT P98,000 x 3.6847) P361,100 Salvage value of new machine, end of 6 years(P20,000 x 0.41044) ( 8,209) Investment in new machine (120,000 – 40,000) 80,000 Total P432,891

29 . Answer: B

511

Page 30: 11 x09 Capital Budgeting

Capital Budgeting

Negative Net Present Value ( 25,310)30 . Answer: B

PV of annuity of 1 at IRR ∑(1 ÷ 1.12386)5 3.57057PV of annuity of 1 at MCC ∑(1 ÷ 1.11055)5 3.69079After-tax cash flows 10,000 ÷ (3.69079 – 3.57057) 83,180.84Investment: 83,180.84 x 3.57057 297,000Profitability index (297,000 + 10,000) ÷ 297,000 1.034A shorter calculation of the Profitability Index can be made by:3.69079 ÷ 3.57057 = 1.034

31 . Answer: DIn discounting the annual cash inflow by the IRR, the NPV = P0The net present value of ZERO is 14% and 16%. For better time management, the candidate is expected not to do detailed calculation of finding out the exact rate.The use of interpolation indicated that the IRR is 15.3%:

Discount RateNet Present Value0.141,197IRR00.16-708(0.14 – IRR) ÷ (0.14 – 0.16) = 1,197 ÷ ( 1,197 + 708)(0.14 – IRR) ÷ -.02 = 1,197 ÷ 1905(0.14 – IRR) ÷ - .02 = 0.628(0.14 – IRR) = 0.628 x -0.020.14 – IRR = 0. 013

IRR = 0.153 or 15.30%Note: Since at the IRR, NPV is zero, the answer can only be between 14% & 16%, since only one of the choices, satisfy the criteria, the answer is (D).

32 . Answer: BThe payback period that corresponds to the project’s internal rate of return of 12 percent is 4.968. Therefore, the amount of investment must equal the product of the payback period and the net cash flows:Investment: (4.968 x 20,000) = P99,360

33 . Answer: DThe amount of investment: the PV of annuity at IRR

4.355 x 6,000 = 26,130

512

Page 31: 11 x09 Capital Budgeting

Capital Budgeting

The manner of financing the project is not considered in the analysis of capital investment. 34 . Answer: C

Present value of cash inflows equals amount of investment at 10% IRR.P20,000 x 3.791 = P75,820

35 . Answer: AATCF: P1,500,000/3.60472 416,121Depreciation 300,000Net income: 416,121 – 300,000 116,121Before-tax income: 116,121/0.60 193,535Fixed costs 500,000Contribution margin: 193,535 + 500,000 693,535Unit sales 693,535 ÷ (100 - 60) 17,338

36 . Answer: BContribution margin (per No. 23) 693,535Divide by sales volume ÷ 20,000Contribution margin per unit P34.68Add variable cost per unit 60.00Selling price per unit P94.68

Alternative Solution:Cash inflow before tax based on present price: (20,000 x 40) – 200,000 600,000After-tax cash inflow (600,000 x 0.6) + (300,000 x 0.4) 480,000Present value of ATCF (480,000 x 3.60478) 1,730,294Investment 1,500,000Net present value (present price) 230,294Annual excess ATCF due to excess price (230,294 ÷ 3.60478) 63,885Before-tax excess cash inflow (63,885 ÷ 0.6) 106,475Excess selling price: 106,475 ÷ 20,000 5.32Reduced selling price to achieve IRR of 12% (100 – 5.32) 94.68

513

Page 32: 11 x09 Capital Budgeting

Capital Budgeting

37 . Answer: CAnnual after-tax cash flow 500,000/5.6502 88,492Depreciation 500,000/10 50,000Net income 38,492Income before tax 38,492/0.6 64,154Depreciation 50,000Cash savings before tax: 64,154 + 50,000 114,154

38 . Answer: AThe amount of annual cash flows can be solved by equation:NPV = PV of annual CF – Investment1,750 = 2.4771CF – 2.4018CF1,750 = 0.1753CFCF = 9,980

39 . Answer: AInvestment 120,000Less Present value of salvage value (12,000 x 0.3855) 4,626Present value of Annual Cash Inflows 115,374Minimum Annual Cash Flows (115,374 ÷ 6.1446) 18,776

40 . Answer: BPresent value of annual cash flows at IRR (81,000 x 4.639) 375,759Investment 81,000 x 4.344 351,864Difference 23,895Annual increase in cash flows 23,895/4.344 5,501

41 . Answer: AInvestment (Total of present value @ IRR of 12%) 50,000Less PV, year 1 & 2 (16,074 + 17,534) 33,608PV of the 3rd cash flow 16,392

514

Page 33: 11 x09 Capital Budgeting

Capital Budgeting

After-tax cash flow, third year 16,392/0.712 23,022

Investment must be separate from financing. It is a normally committed error in the

application of capital budgeting techniques where financing strategy is considered. The explicit or implicit cost of financing the project is taken care of the discounting process.

43 . Answer: BCost of equipment 750,000Less PV of tangible benefits 100,000 x 5.01877 501,877PV of annual intangible benefits 248,123Amount of annual intangible benefits 248,123/5.01877 49,440

44 . Answer: BTo be acceptable, the project should yield a net present value of zero. The negative net present value must be offset by the present value of annual intangible benefits.

Present value of intangible benefits P184,350PV of annuity of 1 at 10% for 10 years ÷ 6.145Annual net intangible benefits P30,000

45 . Answer: AThe indifference rate (crossover or fisher rate) refers to the rate at which the net present values of the 2 alternatives are indifferent or equal.The easier test of the rate is to look for IRR (using trial and error technique) of the investment difference.

Difference 80,000 – 48,000 35,225PV inflows ∑(3,200 ÷ 1.1264)6 (12,922)PV inflows ∑(15,200 ÷ 1.1264)10-6 (22,303) Difference NIL

Alternative Solution:Project XProject YPV of after-tax cash flows

∑(12,000 ÷ 1.1264)648,455 ∑(15,200 ÷ 1.1264)1083,680Investment48,00083,225Net Present Value 455 455

46 . Answer: BThe determination of the indifference point, which is 10%, for the two projects can be made through the use of trial and error estimation.

Machine 1Machine 2PV of Difference in ATCF Year 1 155,000 ÷ 1.10 140,909.10(140,909.10) Year 2 (110,000 ÷ 1.10)2( 90,909.10) 90,909.10Net difference 50,000.00( 50,000.00)Difference in investment( 50,000.00) 50,000.00NPV NIL NIL

515

Page 34: 11 x09 Capital Budgeting

Capital Budgeting

47 . Answer: C15% Discount Rate

Machine 1Machine 2PV of Difference in ATCF Year 1 155,000 x 0.86957 134,783.35(134,783.35) Year 2 110,000 x 0.75614( 83,175.40) 83,175.40Net difference 51,607.95( 51,607.95)Difference in investment( 50,000.00) 50,000.00NPV 1,607.95( 1,607.95) At 15 percent discount rate, Machine 1 is more acceptable.

8% Discount RateMachine 1Machine 2PV of Difference in ATCF Year 1 155,000 x 0.92593 143,519.15(143,519.15) Year 2 110,000 x 0.85734( 94,307.40) 94,307.40Net difference

49,211.75( 49,211.75)Difference in investment( 50,000.00) 50,000.00NPV ( 788.25) 788.25At 8 percent discount rate, Machine 2 is more acceptable.

48 . Answer: CCost of Investment:

Invoice price 950,000Installation cost 24,200Freight charge 800 Total investment 975,000

Annual Cash Flow:Number of procedures: (52 x 5) 260Contribution margin per procedures: (P800 – P10 – P40) P750Total annual cash flow: (260 x P750) P195,000Cash payback period: (975,000 ÷ 195,000) 5 years

49 . Answer: BPresent value of cash flow (195,000 x 6.418) P1,251,510Present value of salvage value (55,000 x 0.42241) 23,233Total P1,274,743Capital investment 975,000Net present value P 299,743

50 . Answer: A

516

Page 35: 11 x09 Capital Budgeting

Capital Budgeting

51 . Answer: A Contribution margin: 300,000 x (75 – 50) 7,500,000Less Fixed costs 4,500,000Cash flow before tax 3,000,000Less: Depreciation (6,000,000 ÷ 4) 1,500,000Income before tax 1,500,000Less: Income tax (1,500,000 x 0.4) 600,000Net income 900,000Add back: Depreciation 1,500,000After-tax Cash Flow 2,400,000

52 . Answer: CPV of After-tax Cash Flows (2,400,000 x 2.9287) 7,028,900Cost of investment 6,000,000Net Present Value 1,028,900

53 . Answer: AAnnual excess present value (1,028,000 ÷ 2.9287) P351,000Excess cash before tax (351,000 ÷ 0.6) P585,000Maximum number of units as decrease (585,000 ÷ 15) 39,000

54 . Answer: AAverage Annual net income:

(100,000 + 40,000 + 20,000 + 10,000 + 10,000) ÷ 5 = 36,000Divide by average investment (400,000 ÷ 2) 200,000Accounting rate of return 18%

Accounting rate of return or unadjusted rate of return computes the profitability of the project in term of accrual profit. Net profit under accrual method considers depreciation, a substantial amount that understates the average profit. This understatement of amount that is used in the computation necessarily requires that preferably, average investment should be used, instead of the initial investment, in the determination of accounting rate of return.

55 . Answer: B

517

Page 36: 11 x09 Capital Budgeting

Capital Budgeting

58 . Answer: DPayback period: Investment ÷ Net Annual Cash InflowP200,000 ÷ P50,000 = 4 years

59 . Answer: DPresent value of Net Cash Inflow (71,000 X 4.355) 309,205Investment 300,000Net Present value 9.205

60 . Answer: BAverage Investment: (200,000 ÷ 2) = 100,000Accounting Rate of Return = Net Income ÷ Average Investment (10,000 ÷ 100,000) = 10 percent

61 . Answer: BThe payback for PA is 4.225. This is closest to the present value of annuity of 1 discounted at 11 percent for 6 periods which is 4.231.

62 . Answer: AAnnual depreciation: (P50,000 ÷ 8) P6,250Annual tax shield: (P6,250 x 0.3) P1,875

63 . Answer: CBefore-tax cash inflow P10,000Less depreciation 6,250Income before tax 3,750Less income tax (3,750 x 0.3) 1,125Net income 2,625Add back depreciation 6,250After-tax cash inflow P 8,875

A quicker calculation of after tax cash flow can be made by adding the tax shield to after-tax cash inflow without any tax benefit on depreciation. (P10,000 × .70) + P1,875 = P8,875

518

Page 37: 11 x09 Capital Budgeting

Capital Budgeting

Cash FlowPV FactorPV of annual net cash flows:180,0000.909163,620120,0000.826 64 . Answer: B

Payback period: (P50,000 ÷ P8,875) = 5.6 years

65 . Answer: CPresent value of annual ATCF (P8,875 x 5.747) P51,000Present value of after-tax salvage value (P1,400 x 0.54) 756 Total 51,756Investment 50,000Net present value P 1,756

66 . Answer: CAt the discount rate of 8 percent, there is a net present value of P1,756. Therefore, the IRR is higher than 8 percent. Using trial and error approach, the first try should use 9 percent. If the present value of the inflows exceeds P50,000, then the IRR is lower than 9 percent, otherwise it should be 9.5 percent.Using 9.0 percent in discounting the inflows, there is a net present value of P(174); therefore the IRR is slightly lower than but very close to 9.0 percent.(P8,875 x 5.535) + (P1,400 x 0.5019) – P50,000 = P(174)

67 . Answer: BAdditional contribution margin:Small 6,000 x 5.40 32,400Medium 15,000 x 6.50 97,500Large 9,000 x 7.90 71,100Total 201,000Less Cash Fixed Expenses:

Rent 18,000Salaries 54,000Utilities 13,200Insurance, etc. 7,800 93,000

Annual Cash Inflows 108,000

68 . Answer: BAdditional rental income 130,000

519

Page 38: 11 x09 Capital Budgeting

Capital Budgeting

99,120100,0000.751 75,10090,0000.683 61,47090,0000.621 55,890Total455,200Amount of investment400,000Net Present Value 55,200

56 . Answer: CPresent Value Index (Profitability Index) Present Value of ATCF ÷ Net Investment (455,200 ÷ 400,000) = 1.14

The present value index computes net present value in terms of P1 investment. Therefore, the index of 1.14 means the net present value per P1 of investment is P0.14. This concept makes the present value index better than the net present value technique because the index indicates which one is the most profitable on a per P1 investment.

Average investment: (975,000 + 55,000) ÷ 2 515,000

Annual depreciation: (975,000 – 55,000) ÷ 10 92,000Annual net income: 195,000 – 92,000 103,000Average annual Rate of Return: P103,000 P515,000 20%

(80,000 x 0.68) + (45,000 x 0.32) 68,800Present value of ATCF (68,800 x 3.23972) 222,893Investment 207,200Net Present Value 15,693

The purpose of profitability index is to compare two projects’ profitability by reducing the present value per 1 peso of investment. Therefore, the ratio of 4.35526 @ 10% to 4.11141 @

69 . Answer: APV of annual cash inflow (108,000 x 5.575) 602,100PV of salvage value (70,000 x 0.108) 3,240PV of working capital return (7,500 x 0.108) 810Total 606,150Investment:

Remodeling cost 550,000Working capital 7,500 557,500

Net Present Value 48,650

70 . Answer: BPV of annual cash inflow (65,000 x 5.575) 362,375PV of salvage value 1,296PV of working capital return 432Total 364,103Investment:

Remodeling cost 290,000Working capital 4,000 294,000

Net Present Value 70,103

520

Page 39: 11 x09 Capital Budgeting

Capital Budgeting

71 . Answer: ARental income 21,000 x 5 105,000Additional cash inflow, snack bar 13,000Total 118,000Less fixed expenses 80,000Annual cash inflow 38,000

PV of annual cash inflow (38,000 x 5,575) 211,850PV of salvage value 1,296PV of working capital return 432Total 213,578Investment 294,000Negative Net Present Value ( 80,422)

72 . Answer: DThe annual cost of advertising can be easily calculated by dividing the net present value of alternative 2, at 16% by the present value of annuity of 1.

70,103 ÷ 5,575 = 12,574.53

73 . Answer: AAnnual revenues 300,000Variable expenses 60,000Contribution margin 240,000Fixed expenses

Advertising 40,000Salaries 110,000Utilities 5,200Insurance 800 156,000

Annual cash income 84,000Less Depreciation 420,000 x 0.90 ÷ 12 31,500Annual Income 52,500

521

Page 40: 11 x09 Capital Budgeting

Capital Budgeting

76 . Answer: BSavings 56,250Less Depreciation 234000 ÷ 13 years 18,000Annual income 38,250Simple Annual Return 38,250 ÷ 225,000 17 %

77 . Answer: CPayback period = Initial Investment ÷ Annual Cash Inflow

420,000 ÷ 84,000 = 5 years

78 . Answer: A225,000 ÷ 56,250 = 4 years

79 . Answer: CPurchase price of new tools 2,500,000Add increase in working capital 60,000Total 2,560,000 Deduct Salvage value of the old tools 45,000Net investment 2,528,500

80 . Answer: CPurchase price of valve stem 80,000 x 20 1,600,000Cost to make:

Direct materials 80,000 x 4.50 360,000Direct labor 80,000 x 3.90 312,000Variable overhead 80,000 x 1.50 120,000

Decrease in directs labor and variable costs 80,000 x 1.60 (128,000) 664,000Cost savings 936,000

81 . Answer: APV of annual depreciation

522

Page 41: 11 x09 Capital Budgeting

Capital Budgeting

82 . Answer: CAfter tax salvage value 100,000 x .7 70,000PV of 1 end of 4 periods 0.63552PV of after – tax salvage value 44,486.4

83 . Answer: CPV of after cash savings 936,000 x .7 x 3.03735 1990072PV of tax benefits from depreciation 603,333PV of after tax salvage value 44,486PV of working capital return 60,000 x 0.63552 38,131Investment (2528,500)Net present value 147,522

84 . Answer: APV of tax benefits, declining - balance 603,333PV of tax benefits, straight-line method 2,500,000 4 x .3 x 3.03735 569,503Net advantage 33,830

85 . Answer: AInvoice price of new equipment (945,000 x 0.98) P926,100Freight 11,000Installation cost 22,900 Total 960,000 Less: Salvage value of old equipment (0.6 x 1,500) 900 Reduction in working capital 2,500 3,400Net initial outflows P956,600

86 . Answer: BTotal variable costs (262,000 units x P20*) P5,240,000Avoidable fixed costs (P45,000 x 5 years) 225,000Total 5,465,000

523

Page 42: 11 x09 Capital Budgeting

Capital Budgeting

87 . Answer: AThe present value of the tax shield based on declining-depreciation is:

YearDepreciationTax Shield (40%)PV FactorPV of Tax Shield2007P319,968P127,9870.893P114,2922008 426,720 170,6880.797 136,0382009 142,176 56,8700.712 40,4922010 71,136 28,4550.636 18,098TotalP308,920

88 . Answer: CPurchase Cost

Year ATCF200750,000 x 27 x 0.6 810,000200850,000 x 27 x 0.6 810,000200952,000 x 27 x 0.6 842,400201055,000 x 27 x 0.6 891,000201155,000 x 27 x 0.6 891,0002006(1,500 x 0.6) ( 900) Total 4,243,500

89 . Answer: APresent value of after-tax cash flows

2007 (810,000 x 0.893) P 723,3302008 (810,000 x 0.797) 645,5702009 (842,400 x 0.712( 599,7892010 (891,000 x 0.636) 566,6762011 (891,000 x 0.567) 505,197Salvage value of old equipment (1,500 x 0.60) (900)Net present value P3,039,662

90 . Answer: DCFBTCFATPV FactorPVCFAT2006Initial outflow(P956,600)2007(50,000 x 20) + 45,000

(1,045,000 x 0.6) - (319,968 x 0.4)1,045,000499,013

0.893445,6192008(1,045,000 x 0.6) – (426,720 x 0.4)456,3120.797363,6812009(52,000 x 20) + 45,000

(1,085,000 x 0.6) – (142,176 x 0.4)1,085,000

594.1300.712

423,0212010(55,000 x 20) + 45,000(1,145,000 x 0.6) – (71,136)1,145,000

658,546

524

Page 43: 11 x09 Capital Budgeting

Capital Budgeting

PeriodDepreciationPV FactorPresent ValueYear 1 832,5000.89286743,305.95 2 112,5000.79719886,873.88 3 370,0000.71178263,358.60 4

185,0000.63552117,571.20Total2,011,109.63Tax rate0.30PV of tax benefits from depreciation603,332.89

74 . Answer: ACurrent operating costs – old machine 78,000Deduct Operating costs – Machine B

Annual salary of operator 16,350Annual maintenance cost 5,400 21,750

Annual cash savings 56,250

75 . Answer: ASimple Rate of Return = Net Income ÷ Initial Investment

52,500 ÷ 420,000 = 12.50 %

12% indicated the profitability index. Profitability index: 4.35526/4.11141 = 1.06

42 . Answer: BThe net present value = PV of excess salvage value less PV of decrease in after-tax cash flow

Let X = the excess salvage value7,003 = 0.56743X – [3.60478 x (0.2X * 0.4)7,003 = 0.56743X – 0.2883824X7,003 = 0.2790476X X = 25,096

Required salvage value: 50,000 – 25,096 = 24,904

57 . Answer: D Cash InflowUnrecovered InvestmentPeriod 0 Outflows(400,000)Period 1180,000(220,000)Period

2120,000(100,000)Period 3100,000Zero

The total outflows are fully recovered by the end of period 3.The analyst should be careful in computing the payback period when the project has uneven cash inflows. The common error in handling uneven cash flows is using the average cash flows instead of reducing the unrecovered outflows.

Additional cash flow, snack bar 15,000Total 145,000Less Cash Fixed Expenses:

Rent 48,000Salaries 17,000Utilities 5,400Insurance, etc. 9,600 80,000

Annual Cash Inflow 65,000

After-tax Cash outflowsOperating expenses (5,465,000 x 0.6) P3,279,000Depreciation (960,000 x 0.4) ( 384,000)After-tax salvage value of new equipment (12,000 x 0.60) ( 7,200) Net outflows P2,887,800

*Variable cost per unitDirect material (10.00 x 0.75) P 7.50Direct labor 8.00Variable overhead (6.00 x 0.75) 4.50 Total P20.00

0.636418,8352011(55,000 x 20) + 45,000

(1,145,000 x 0.6)1,145,000687,000

0.567

525

Page 44: 11 x09 Capital Budgeting

Capital Budgeting

385,447Salvage value (12,000 x 0.6)7,200P2,993,203

526


Recommended